Finding Derivitive with Expression of Area

  • Thread starter Thread starter odolwa99
  • Start date Start date
  • Tags Tags
    Area Expression
Click For Summary
The discussion centers on solving a problem involving the differentiation of an expression related to the surface area of a tank shaped like a cylinder with a hemispherical top. The user successfully expressed the height of the cylinder in terms of the radius but encountered difficulties in differentiating the expression to find maximum volume. The attempt to find the derivative led to incorrect conclusions about the values of r, indicating a misunderstanding of the problem's requirements. It was clarified that the formula being maximized should pertain to volume, not surface area. The correct radius value from the textbook is provided as r = 3.
odolwa99
Messages
85
Reaction score
0
I have managed to solve (i), so I'll just post the answer as it comes into play for (ii). I'm struggling with the differentiation of this expression. Can anyone help?
Many thanks.

Homework Statement



Q. A tank, with a base, is made from a thin uniform metal. The tank, standing on level ground, is in the shape of an upright circular cylinder & hemispherical top, with radius length of r metres. The height of the cylinder is h metres. (i) If the total surface area of the tank is 45∏m2, express h in terms of r, (ii) Find the values of h & r, for which the tank has maximum volume.

Homework Equations





The Attempt at a Solution



Attempt: (i) \frac{45 - 3r^2}{2r}

(ii) 1st, separate the fractions and simplify the answer in (i) to \frac{45}{2r} - \frac{3r}{2}
\frac{dS}{dx} = -\frac{45}{r^2} - \frac{3}{2} = 0 => \frac{3r^2}{2} = -45 => 3r2 = -90 => r2 = -30 => r = -\sqrt{30}

Ans: (From textbook): r = 3
 
Physics news on Phys.org
Your formula you are trying to maximize is that of surface area, not volume!
 
Question: A clock's minute hand has length 4 and its hour hand has length 3. What is the distance between the tips at the moment when it is increasing most rapidly?(Putnam Exam Question) Answer: Making assumption that both the hands moves at constant angular velocities, the answer is ## \sqrt{7} .## But don't you think this assumption is somewhat doubtful and wrong?

Similar threads

  • · Replies 2 ·
Replies
2
Views
1K
  • · Replies 7 ·
Replies
7
Views
1K
  • · Replies 9 ·
Replies
9
Views
2K
  • · Replies 18 ·
Replies
18
Views
2K
  • · Replies 33 ·
2
Replies
33
Views
3K
  • · Replies 3 ·
Replies
3
Views
2K
  • · Replies 2 ·
Replies
2
Views
2K
Replies
3
Views
2K
  • · Replies 10 ·
Replies
10
Views
2K
  • · Replies 8 ·
Replies
8
Views
2K